Peds the point NCLEX style The point E1

अब Quizwiz के साथ अपने होमवर्क और परीक्षाओं को एस करें!

The nurse is conducting a physical examination of a young preschooler and detects the odor of tobacco smoke on the parents' hair and clothing. How should the nurse respond? A) "Are you aware that exposure to tobacco smoke can be associated with behavioral difficulties?" B) "Tobacco smoke is linked with an increased number of ear infections." C) "You should stop smoking around your child." D) "Tell me about your child's exposure to tobacco smoke."

ANSWER D The nurse needs to emphasize that parents should protect their child from all tobacco smoke. It is best to ask an open-ended question to determine who uses tobacco products or where exposure to tobacco smoke could be occurring. The nurse should not assume that the parents are smokers and telling them to stop is inappropriate. Telling the parent tobacco smoke can be linked to ear infection and behavioral difficulties is true but is unlikely to open a dialogue.

A parent expresses surprise to the nurse that her toddler daughter has begun masturbating. The most important initial nursing response is: A)This is a normal and expected activity best treated matter-of-factly. B)Toilet teaching places much focus on the genitals. C)Toddler girls as well as boys will masturbate. D)Check for undue stress in your toddler's life.

ANSWER A Masturbation is a normal event to be done in private. Calling attention to the behavior may increase the frequency. Both girls and boys masturbate, and toilet teaching calls attention to the genital area. These two statements are accurate information but not the best first response. Excessive or public masturbation points to stress.

The nurse is teaching the parent of a 2-year-old child about age-appropriate toys. Which would be of most interest plus stimulating to the growth and development of this child? A)Giving the child bowls, pot, pans, and large spoons B)Providing a brightly colored plastic bucket and shovel C)Offering the child a variety of large stuffed toys D)Giving the child a toy vacuum cleaner

ANSWER A The kitchen items are usually of most interest since they give opportunity to copy observed parental actions. Also, these items can be used not only to role model but also to stack, nest, make noise, and rearrange in many configurations. They are also inexpensive. However, all the other toys are appropriate and safe for toddlers.

The parent of a 4½-year-old boy has contacted the nurse because he is concerned that his son is frequently touching his genitals. The nurse explains that this is normal during the preschool years. Which statement by the parent would indicate a need for further teaching? A)"I will need to find an appropriate punishment for him if this continues." B)"This is probably a good time to talk about safety and who can touch him." C)"I should teach him certain rules about this activity." D)"I have heard that both boys and girls often touch themselves at this age."

ANSWER A The nurse should remind the parent that overreaction to this behavior may cause it to occur more frequently. Masturbation at this age should be treated matter-of-factly. The other statements are correct.

The parents of a 4-day-old infant report concern about his weight loss. What is the best response by the nurse? A)"With appropriate nutrition weight gain will commence with a return to the birth weight within 2 weeks." B)"Babies will begin to rapidly regain weight and will double birth weight around 4 to 6 months of age." C)"Weight loss after birth is normal." D)"Babies may lose up to 10% of their body weight in the first month of life."

ANSWER A The question asks for the best response which typically informs or instructs the client on a situation in which they are concerned. The average newborn weighs 7 lb 8 oz (3,400 g) at birth. Newborns lose up to 10% of their body weight over the first week of life. The average newborn then gains about 30 g per day and regains his or her birth weight by 10 to 14 days of age. Most infants double their birth weight by 4 to 6 months of age and triple their birth weight by the time they are 1 year old.

The nurse finds the diet of a 30-month-old child to be low in calcium. What suggestion can significantly increase this toddler's calcium intake? A)Give her slices of cheddar cheese as a snack. B)Use unsweetened applesauce as a dessert. C)Offer chocolate milk to increase milk intake. D)Include dark greens and spinach in her meals.

ANSWER A Two and one-half ounces of cheddar cheese provides the toddler's daily requirement of 500 mg of calcium. Chocolate milk provides calcium but the sugar it contains should not be a regular part of a toddler diet. Applesauce provides fiber, not calcium. Spinach and dark greens do contain calcium, but that calcium has limited bioavailability.

The nurse is assessing a 2-year-old toddler. Which observations would alert the nurse that the child may be developmentally delayed? (Select all that apply.) A)The child will not pick up a toy or touch the nose when directed by the nurse. B)The child pushes and pulls the play vacuum cleaner in the toy room. C)The child claps hands in response the nurse clapping hands. D)The child's vocabulary consists of the words "ball," "dada," "mum," "drink" and "up." E)The child is unable to stack one block on top of another.

ANSWER A-D The 2-year-old toddler should be speaking in 2-word sentences, and should be able to follow simple commands. Imitating the clapping of hands and pushing and pulling objects is expected of a 2-year-old toddler. The child should not be able to stack blocks until the age of 3, so this observation would not be a concern at this age.

The nurse is reviewing the diet of an 8-month-old infant with the mother who reveals she has been using evaporated milk to make the formula. Which additional ingredient should the nurse ensure she is including in the formula? A)Vitamin D B)Iron C)Calcium D)Vitamin E

ANSWER B

A new mother reports that she is exhausted and that the little sleep she gets is determined by her baby. Which suggestion should the nurse prioritize to help the mother establish healthy sleeping patterns in her infant? A)Wake the baby from afternoon naps after 1 hour to ensure she is sleepy by nighttime. B)Use the crib for sleeping only, not for play activities. C)Put the baby to bed at various times of the evening. D)Let the baby cry during the night and she will eventually fall back to sleep.

ANSWER B A consistent bedtime routine is usually helpful in establishing healthy sleeping patterns and in preventing sleep problems. Using the crib for sleeping only helps the child associate the bed with sleep. Depriving the baby of sleep during the afternoon or evening will make the baby over-tired and less able to establish a healthy sleeping pattern. While letting a baby cry for a while is acceptable, this does not promote consistency in the baby's sleeping pattern.

The parent of an infant questions the nurse about the baby's teething. The nurse provides client education. Which statement by the parent indicates understanding of the information provided? A)"By 1 year my baby should have about three teeth." B)"The first teeth that will likely appear are the lower incisors." C)"My baby will most likely have the upper middle teeth come in first." D)"My baby's first tooth will likely appear between 5 and 6 months."

ANSWER B Teeth will begin erupting between 6 and 8 months. Traditionally, the first teeth to erupt will be the lower incisors, followed by the upper incisors. By the age of 12 months, the infant will have between 4 and 8 teeth, if progressing normally.

In assessing the bilingual 4-year-old's speech and language development, the nurse should find that the child will: A) mix the two languages in the same sentence. B)be able to use each language as a separate system. C)be prone to stuttering. D)avoid asking questions.

ANSWER B The ability to use both languages separately is developed by 4 years. Mixing the two languages occurs during toddlerhood. The other options are not related to bilingualism and are incorrect.

The nurse is examining a 3-year-old child during a regular visit. Which finding would disclose a developmental delay in this child? A)The child imitates the nurse in use of a stethoscope. B)The child demonstrates separation anxiety. C)The child follows directions when made one at a time. D)The child copies a circle on a piece of paper.

ANSWER B The child should be past the stage of separation anxiety by age 3 years. Imitating actions, copying a circle on paper, and responding to single requests are developmentally appropriate.

In counseling a group of parents regarding the development of school readiness in preschoolers, the nurse emphasizes: A)library visits and story hour. B)the home environment. C)visiting the kindergarten. D)enrollment in a preschool.

ANSWER B The home environment is the foundation for the child's learning and is the most important element for developing school readiness. Values regarding learning originate at home. Parents are the first teachers and role models. Structure, proper behavior with others, and language skills are all developed at home and are essential for school success. The other options are important but not as vital as the home environment. A preschool experience can foster social skills and group interaction. Library story hour and books develop language skills, and visiting a kindergarten can help the child make the transition to school.

A 3½-year-old shouts, "Look out for Boo-ga-loo!" as the nurse enters the exam room. The parent explains Boo-ga-loo is his daughter's imaginary friend. How should the nurse respond? A)"Where did you get that funny name?" B)"Tell me about Boo-ga-loo." C)"Are you kidding me?" D)"I don't see anyone."

ANSWER B The imaginary friend serves as a companion and playmate for the preschooler. The child knows this friend is not real, yet is very invested in the imaginary playmate who can be and do anything the child's fantasy life can invent. The nurse should acknowledge this friend, not minimize the friend's importance. "Tell me..." does this. The other responses do not.

The parent of 1-week-old infant voices concerns about the infant's weight loss since birth. At birth the infant weighed 7 lb (3.2 kg); the infant currently weighs 6 lb 5 oz (2.9 kg). Which response by the nurse is most appropriate? A)"Your infant has lost a bit more than the normal amount." B)"Your infant's weight loss is within the expected range." C)"Your infant has lost too much weight and may need to be hospitalized." D)"All infants lose a substantial amount of weight after birth."

ANSWER B The normal newborn may lose up to 10% of birth weight. This infant has lost 9.1%. This degree of weight loss will likely not require hospitalization. Expressing to the parent that the infant may be hospitalized is rash and will most likely not occur.

The nurse is caring for a 4-year-old child following an appendectomy. The child becomes fearful and starts to cry as soon as the nurse walks into the room. When the nurse asks about the crying, the child says, "Nurses who wear shirts with flowers give shots." The nurse understands that this statement is an example of: A) animism. B) transduction. C) beginning empathy. D) magical thinking.

ANSWER B The nurse identifies transduction. Because the 4-year-old recently received an injection from a nurse in a flowered uniform, the child believes that all nurses who wear flowered uniforms give shots. Transduction is reasoning by viewing one situation as the basis for another situation even though the two may or may not be causally linked. Magical thinking involves believing that one's thoughts are all-powerful. Animism is attributing life-like characteristics to inanimate objects. Empathy is the understanding of others' feelings.

The nurse is providing teaching about preventing poisoning. Which statement by the parent would warrant further discussion? A)"I never transfer cleaning products out of their original containers." B)"All medicine in our bathroom is in childproof containers." C)"We keep our lawn and garden products on a high shelf in the garage." D)"We have the poison control number located near our phone."

ANSWER B The nurse should emphasize that while childproof caps on medications are important, all medications including those with childproof caps should still be kept locked. (Vitamins are medications.) The other statements are correct.

A male child exhibits the following growth pattern. How should the nurse interpret these data? 3 yrs: Ht. 37 in Wt. 32 lb 4 yrs: Ht. 39 in Wt. 39 lb 5 yrs: Ht. 40 in Wt. 46 lb A) The child may be at risk for hyperpituitarism B) The child is growing slowly in height but rapidly in weight. C) The child is following a normal pattern of growth for the preschool years. D) The child's height and weight should be plotted on a growth chart.

ANSWER B The preschool child's growth is fairly even. The child should grow about 2½ to 3 in and gain around 5 lb yearly. This child is not following this pattern. The child's pattern is growing slowly in height but rapidly in weight. Thus, the child is at risk for becoming overweight, not hyperpituitarism which is production of excessive growth hormone. Plotting the child's height and weight on a growth chart would make it visually easy to follow his growth pattern and compare it to the norms.

The nurse is helping the mother of a 5-month-old boy understand the importance of developmentally appropriate play. Which one of the toys best meets the needs of this child? a)A push-pull toy b)A yellow rubber duck for the bath C)Pots and pans from the kitchen cupboard d)Brightly colored stacking toy

ANSWER B The rubber duck is most appropriate. It is safe, visually stimulating while bobbing on the water, and adds pleasure to bath time. A push-pull toy promotes skill for a walking infant. Pots and pans from the kitchen cupboard are played with successfully after sitting is mastered. A 5-month-old does not have the fine motor coordination to use stacking toys.

Nurses should provide anticipatory guidance to males to prepare them for what particular pubertal change in late adolescence? a)Reddening of the scrotum B)Nocturnal emissions C)Breast enlargement D)Lengthening of the penis

ANSWER B This involuntary ejaculation during the night can be disturbing to the adolescent male who has little or no understanding of what is happening in his body. Lengthening of the penis begins to occur in early adolescence as does reddening of the scrotum. Breast enlargement occurs in some males in middle adolescence and resolves in late adolescence.

A young breastfeeding parent calls the telephone nurse because the parent is concerned about the 3-month-old's stools. Which information indicates a possible problem? A"The infant hasn't had a stool for 3 days." B"The stools are foamy and smell terrible." C"The infant's stools are loose and seedy." D"The infant grunts and squirms when filling the diaper.

ANSWER B This may indicate a digestive problem or illness. The health care provider or nurse practitioner should be contacted. All the other statements describe normal stooling.

Parents of a toddler describe how they handled their child's temper tantrum in a shopping mall. What action of the parents indicates need for additional teaching? A)Made sure the child was rested and not hungry before going to the mall B)Reasoned with the child to stop the behavior C)Remained relatively calm even though embarrassed D)Tried to refocus the child's attention as tantrum behavioral cues appeared

ANSWER B The child having a tantrum is out of control, making reasoning impossible. Calmly bear hugging the child provides control, especially in a public place. The other actions are helpful in preventing a tantrum.

The nurse is promoting a healthy diet to the parent of a 6-month-old infant. What action would have the most effect on the infant's neurologic development? A)Requiring more solid foods in the diet B)Establishing an adequate level of dietary iron intake C)Promoting continuation of breastfeeding D)Adding fruit juice daily

ANSWER C Continuing to breastfeed ensures the proper level of nutritional fat for myelination of the nervous system. Having adequate dietary iron would help prevent anemia as the stores from fetal development are depleted. Promoting increased intake of solid foods is not necessary at 6 months and may diminish the amount of breast milk consumed. Fruit juice in the diet is not recommended. Fruits provide more nutrition and will soon be gradually added to the infant's diet.

The nurse is promoting language and cognitive development to the parents of a 3-year-old child. Which guidance about reading with their child will be most helpful? A)Keep story time a reward for being good. B)Read a different book if he knows the story. C)Ask the child questions as you read. D)Have the child sit still during the story.

ANSWER C Engage the child by asking him questions as he listens. This gives him a chance to contribute to the story. The child does not have to sit still. He may want to move around or even act out part of the story. Story time should happen regularly and not be just a reward. Even if the child can tell the story, he may wish to hear it read again because he enjoys the repetition and familiarity.

The nurse is examining an 8-month-old child for appropriate development during a regular check-up. Which observation points to a developmental risk? A)Picks up small objects using entire hand B)Crawls with stomach down C)Uses only the left hand to grasp D)Cannot pull self to standing

ANSWER C Favoring one hand over the other may be a warning sign that proper motor development is not occurring in the other arm or hand. Grasping small objects with the entire hand is common at 8 months and precedes the pincer grasp, which is used about 2 months later. Crawling with stomach down and being unable to pull to standing are abilities that may not occur for another 4 to 8 weeks.

In discussing with the nurse their 2-year-old's behavior, which of the parents' descriptors suggests the child may be ready for toilet teaching? A)The child frequently repeats words parents just said. B)The child often removes her shoes and socks. C)The child hides behind her bedroom door when defecating. D)The toddler walks with a wide, swaying gait.

ANSWER C Hiding while defecating indicates awareness of this need. Repeating words promotes language development but doesn't indicate readiness for toilet teaching. Walking with a wide, swaying gait is early walking behavior. Steady walking and running signal toileting readiness. Removal of shoes and socks is easily done. Greater fine motor clothing removal skill is needed for toileting.

The parent of 1-week-old infant voices concerns about the infant's weight loss since birth. At birth the infant weighed 7 lb (3.2 kg); the infant currently weighs 6 lb 5 oz (2.9 kg). Which response by the nurse is most appropriate? A)"Your infant has lost a bit more than the normal amount." B)"All infants lose a substantial amount of weight after birth." C)"Your infant's weight loss is within the expected range." D)"Your infant has lost too much weight and may need to be hospitalized."

ANSWER C The normal newborn may lose up to 10% of birth weight. This infant has lost 9.1%. This degree of weight loss will likely not require hospitalization. Expressing to the parent that the infant may be hospitalized is rash and will most likely not occur.

The nurse is playing a game with a toddler in the hospital room. What is the most important benefit of this nurse--client interaction? A)Occupying the toddler's time while the parents are meeting with the health care provider B)Utilizing clinical time when the unit is not busy C)Developing a trusting relationship with the nurse D)Distracting the toddler from expending too much energy with active playing

ANSWER C While all of these are benefits of playing a game with the child, the most important benefit is establishing a relationship with the nurse. This will help the child to be less fearful in an unfamiliar environment and foster a better treatment and recovery period while hospitalized.

A male child exhibits the following growth pattern. How should the nurse interpret these data? 3 yrs: Ht. 37 in Wt. 32 lb 4 yrs: Ht. 39 in Wt. 39 lb 5 yrs: Ht. 40 in Wt. 46 lb Unanswered optionThe child's height and weight should be plotted on a growth chart. Unanswered optionThe child may be at risk for hyperpituitarism Unanswered optionThe child is following a normal pattern of growth for the preschool years. Correct optionThe child is growing slowly in height but rapidly in weight.

ANSWER D The preschool child's growth is fairly even. The child should grow about 2½ to 3 in and gain around 5 lb yearly. This child is not following this pattern. The child's pattern is growing slowly in height but rapidly in weight. Thus, the child is at risk for becoming overweight, not hyperpituitarism which is production of excessive growth hormone. Plotting the child's height and weight on a growth chart would make it visually easy to follow his growth pattern and compare it to the norms.

The nurse is organizing an indoor play area for preschoolers. What play materials are least important? A)Dress-up clothes and hats and puppets B)Cut, paste, drawing, and painting materials C)Balls, blocks, and puzzles D)Electronic teaching toys

ANSWER D These are expensive and usually cannot be used in multiple ways or shared with a group of children. All the rest of the items have many uses, stimulate imagination and creativity, promote social interaction, and are relatively inexpensive and readily replaced.

The parents of a 4-day-old infant report concern about the infant's weight loss. What is the best response by the nurse? A)"Babies may lose up to 10% of their body weight in the first month of life." B)"Babies will begin to rapidly regain weight and will double birth weight around 6 months of age." C")Weight loss after birth is normal." D)"With appropriate nutrition weight gain will commence with a return to the birth weight within 2 weeks."

Answer D Rationale: The average newborn weighs 7 lb 8 oz (3,400 g) at birth. Newborns lose up to 10% of their body weight over the first week of life. The average newborn then gains about 30 g per day and regains his or her birthweight by 10 to 14 days of age. Most infants double their birth weight by 4 months of age and triple their birth weight by the time they are 1 year old.

The nurse is teaching the parent of a 2-month-old infant about the social and emotional developments that will occur in the next 8 weeks. Which behavior is most likely to occur? A)Becoming clingy around strangers B)Participating in a game of peek-a-boo C)Mimicking parent's facial expressions D)Crying when the parent is out of sight

Answer c Infants will mimic the facial expressions of their parents when they are 3 to 4 months old. Becoming clingy around strangers probably won't occur until the child reaches 6 months. Engaging in peek-a-boo becomes fun between 6 and 8 months. Crying when the parent is out of sight indicates separation anxiety and is common after 6 to 8 months of age.

A family of five seeks care for their preschooler with an upper respiratory infection. The facility has no medical record for the family. Why does the nurse encourage this family to establish a medical home? To obtain improved health insurance coverage To receive priority treatment in urgent situations To ensure all the children receive low-cost immunizations To establish a continuing relationship with a healthcare provider or nurse practitioner

Rationale: A medical home is a healthcare provider or nurse practitioner with a long-term, comprehensive relationship with the family. This results in better health supervision and overall improved care. Having a medical home does not give special treatment to its constituents in the way of priority or services, insurance coverage, or cost reduction, although, because of the high level of health supervision, some of these benefits could result due to illnesses being prevented or discovered and treated early

The nurse is examining the back and spinal area of a 14-year-old client. A small dimple is noted. What action is most appropriate? Explain to the parents and child that additional studies may be needed to determine if there is cause for concern about the dimple. Question the teen about muscle weakness in the lower extremities. Assess the hips for symmetry. Document the finding as normal.

Rationale: A normal pilonidal dimple is sometimes seen at the base of the spine. This finding should be documented. There is no additional study or evaluation of this area indicated at this time. Muscle weakness and asymmetry are not associated with the presence of the dimple which is benign.

Which question by the nurse is the best one to elicit complete information about a young client's immunization status? "Tell me which immunizations your child needs today." "When and where did your child receive his last immunization?" "Are your child's immunizations up-to-date?" "Do you have any questions about the immunizations children need at various ages?"

Rationale: The when/where questions gather relevant information and are good starting points for further investigation of the immunization status as well as an opening for discussion of any concerns. The parent is likely to be able to answer these questions. Asking which immunization the child needs and questions regarding immunizations at various ages may cause the parent to be unable to answer and create discomfort. The up-to-date question will likely result in a "yes-no" response and yield little information and not further discussion.

The healthcare provider has made a notation in the medical record of a 17-year-old adolescent that the client is not demonstrating successful completion of the Erikson stage of development. What behavior would be consistent with this assessment? The adolescent is distrustful of others. The adolescent is sexually promiscuous. The adolescent is anxious to move away from the parent's home. The adolescent is uncertain and frequently unable to make decisions.

Rationale: According to Erickson's stages of development the teen develops a sense of identity. Failure to successfully complete this stage will result in a lack of self-confidence and an inability to see oneself as an independent being. The establishment of the ability to trust is completed in an earlier stage of psychosocial development. A desire to move away from the parental home is not uncommon and is not a sign of impaired navigation of this level of psychosocial development.

A 14-year-old client has been brought to the pediatric ambulatory care clinic for a "sports physical" by her parent. The adolescent tells the nurse about not wanting to have the parent present during the examination. What action by the nurse is most appropriate? Explain to the client that since being under the age of 16 means the examination must occur with a parent in attendance. Ask the parent to wait in a separate area nearby until the physical examination has been completed. Talk with the adolescent and the parent together to ask for each to sign a consent waiver for this request to be granted. Ask the healthcare provider for permission to proceed with the client's request.

Rationale: Adolescents may be modest and uncomfortable having a physical examination in front of their parents. When possible requests by adolescents for privacy should be granted.

Due to a certain warning sign, the nurse is anticipating that health supervision for a 7-year-old child will be challenging. Which indicator supports this concern? The family maintains a large garden. Older grandparents play a significant role in the family. The child has regular chores and responsibilities at home. The parents made several negative remarks about the child.

Rationale: Disparaging remarks about the child is a warning sign. Lack of respect for the child can undermine the nurse-parent-child partnership needed for successful health supervision. Older grandparents who follow a healthy lifestyle are a plus for the child. Developmentally appropriate home responsibilities suggest positive parenting practices. A garden providing fresh produce can support good eating habits

A single parent has brought her 9-month-old, recently adopted Chinese child for a health supervision visit. Although there are screening documents from China and the child seems healthy, the nurse plans to screen for infectious diseases. What explains the nurse's caution? Many babies adopted from foreign countries have pediculosis. Testing by the child's home country is unreliable. The child may have come from rural China. Infants tend to have insidious symptoms.

Rationale: Documents from many foreign countries have proven unreliable. Universal screening is recommended for internationally adopted children. Insidious symptoms are common to infants overall. Pediculosis is not an infectious disease (lice do not carry disease). Internationally adopted children generally come from areas with prevalence of infectious disease, so having come from a rural area is not a particular risk indicator.

The nurse is examining the testicles of a 6-year-old client. How can the nurse prevent a retractile testis from slipping back up the inguinal canal? Place one finger over the inguinal canal. Ask the client to stand. Apply gentle pressure on the inguinal canal. Ask the client to sit cross-legged.

Rationale: For a 6-year-old client, sitting cross-legged reduces the cremasteric reflex that retracts the testicles during palpation. Having a client stand is best for an adolescent. Placing a finger over the inguinal canal or applying gentle pressure would be best for an infant.

Parents comment that their child seemed more coordinated on the basketball floor at 12 years than now at 14. The nurse can include what information in the response? Hand--eye coordination is affected negatively by puberty. It is more difficult to keep focused at 14 years than at 12. Rapid and uneven growth of early adolescence (11 to 14 years) can interfere with coordination. This may be a reaction to adult authority.

Uneven growth of soft tissues and bones during growth spurts can cause decreased coordination for boys. The age of 14 years is usually the time of peak height velocity (PHV). Mental focus and reaction to adult authority are not causes of poor coordination. Gains in hand—eye coordination are made during puberty and become precise by late adolescence.

Parents who just moved into their "dream home" are concerned because their toddler, who had achieved daytime bowel and bladder control, has begun wetting and defecating in the underwear. The nurse explains this is called: A)Regression B)Ritualism C)Autonomy D)Egocentrism

ANSWER A He is returning (regressing) to an earlier, possibly safer stage of development caused by a stressor (moving). Autonomy is a stage of development in Erikson's theory. Egocentrism refers to the toddler's focus on self. Ritualism for toddlers provides sameness, the expected, and security.

At birth the newborn's head and chest circumference were measured. The nurse knows that the head should be about: A)½ in smaller than the chest. B)equal in size to the chest. C1 in larger than the chest. D)2 in larger than the chest.

ANSWER C The head is about 1 inch (2.5 cm) larger than the chest and will grow rapidly during the first 6 months. Chest circumference is not routinely measured after the newborn period.

The nurse is providing parental anticipatory guidance to promote healthy emotional development in a 12-month-old child. Which statement best accomplishes this? A)Aggressive behaviors such as hitting and biting are common in toddlers. B)A regular routine and rituals will provide stability and security. C)A sense of control can be provided through offering limited choices. D)Emotions of a 12-month-old are labile. He can move from calm to a temper tantrum rapidly.

ANSWER b Toddlers benefit most from routines and rituals that help them anticipate events and teach and reinforce expected behaviors. Knowing that a child can move from calm to temper tantrum very quickly, understanding the benefit of limited choices, and realizing that hitting and biting are common behaviors in toddlerhood provide information but not a guiding concept.

The nurse is helping the parent of a 5-month-old infant understand the importance of developmentally appropriate play. Which one of the toys best meets the needs of this child? A)A push-pull toy B)Pots and pans from the kitchen cupboard C)A yellow rubber duck for the bath D)Brightly colored stacking toy

Answer C The rubber duck is most appropriate. It is safe, visually stimulating while bobbing on the water, and adds pleasure to bath time. A push-pull toy promotes skill for a walking infant. Pots and pans from the kitchen cupboard are played with successfully after sitting is mastered. A 5-month-old does not have the fine motor coordination to use stacking toys.

CHAPTER 27

CHAPTER 27

The nurse is assessing a 6-month-old infant in the clinic. Which characteristic represents normal language development for this age? A)producing noises when spoken to B)babbling C)Laughing out loud D)Cooing

ANSWER B Cooing begins in the first 4 weeks of life, productions of noises when spoken to and laughing out loud are seen later than 6 months of age. Infants begin to babble around 6 months of age.

The nurse is discussing proper discipline with the parent of a 15-month-old child. Which statement is most important? A)Toddlers are unable to learn rules easily. B)Rules and limits should be simple and few. C)Never spank the child for any reason. D)Use praise when the child is doing something right.

ANSWER C Toddlers younger than 18 months of age should not be spanked due to the possibility of injury. All the other statements describe toddler characteristics accurately and are basic to good discipline but are not the most important for this young toddler.

The nurse is assessing development of a 4-month-old infant during a well-child visit. Which observation needs further investigation? A)The infant shows interest in looking at near or high-contrast objects. B)The infant turns the head in the direction of a squeak toy. C)The infant makes babbling sounds, coos, and smiles. D)The infant responds to the parent when the infant sees him or her but not at other times when the parent is near.

ANSWER D If the infant does not respond to the parent's voice, it could indicate hearing loss. Infants recognize parents' voices from 1 month of age. It is normal for the infant to turn the head in the direction of a squeak toy, to focus visually on near or high-contrast objects, and to make babbling sounds but no words by this age. Infants develop a social smile at 2 months.

The infant measures 21.5 in (54.6 cm) at birth. If the infant is following a normal pattern of growth, what would be an expected height for the infant at the age of 6 months? A)29 in (74 cm) B)32 in (81 cm) C)30.5 in (77.5 cm) D)27.5 in (70 cm)

ANSWER D Infants gain about 0.5 to 1 in (1.25 to 2.5 cm) in length for each of the first 6 months of life. Therefore, a 21.5-in (54.6-cm) infant adding 6 in (15 cm) of growth would be 27.5 in (70 cm). Infants grow the fastest during the first 6 months of life and slow down the second 6 months. By 12 months of age, the infant's length has increased by 50%, making this infant 32 in (81 cm) at 1 year old.

The nurse conducting a 6-month well-baby check-up assesses for the presence/absence of the asymmetric tonic neck reflex. At this age the reflex: A)is expected to appear within 1 month. B)should be pronounced and easy to elicit. C)is a protective reflex and retained for life. D)should have disappeared.

Answer D Rationale: This primitive (not protective) reflex should be present at birth and disappear around age 4 months.

The nurse is caring for a 5-week-old infant who is spitting up "all the time." This is the parent's first child. What should be the priority nursing intervention? A)Offer assurance that spitting up is normal. B)Observe the parent during feeding and burping the infant. C)Describe the capacity of a 5-week-old infant's stomach. D)Recommend the parent offer smaller and more frequent feedings.

Answer b Rationale: Assessing the parent's feeding and burping technique is the first nursing action needed. The parent may be overfeeding or inadequately burping the child. Recommending smaller and more frequent feedings would be determined by the assessment. Assuring the parent that some spitting up is normal and describing the capacity of the infant's stomach is helpful information but not the priority.

The nurse is examining an 8-month-old child for appropriate development during a regular check-up. Which observation points to a developmental risk? A)Crawls with stomach down B)Cannot pull self to standing C)Uses only the left hand to grasp D)Picks up small objects using entire hand

Answer c Rationale: Favoring one hand over the other may be a warning sign that proper motor development is not occurring in the other arm or hand. Grasping small objects with the entire hand is common at 8 months and precedes the pincer grasp, which is used about 2 months later. Crawling with stomach down and being unable to pull to standing are abilities that may not occur for another 4 to 8 weeks.

Which activity is most beneficial in the development of the newborn? A)being sung to by his mother B)listening to classical music C)laying on his back with a mobile overhead D)placement in an infant swing in a position to allow observation of the family's activities

ANSWER

The nurse pulls the 5-month-old to sitting position from supine and notes head lag. The nurse's response is to: rA)efer the infant for developmental and/or neurologic evaluation. B)consider this a normal response for the age. C)suggest more awake tummy time for the child. D)conclude the earlier assessments carried out fatigued the infant.

ANSWER A There should be no head lag by 4 months. Head lag in the 5-month-old may indicate motor or neurologic problems and needs immediate follow-up. All other nursing actions indicate failure to recognize the problem.

The infant in the exam room has these signs and symptoms. Which will the nurse attribute to teething? (Select all that apply.) A)Refusing to eat B)Increased sucking on hands C)Drooling and biting D)Irritability and awakening from sleep E)Fever and diarrhea

ANSWER A-B-C-D Fever and diarrhea are considered signs of illness, not teething. The others are typical of teething.

The nurse is providing teaching about car seat safety for a parents' meeting at the preschool their children attend. Choose the points the nurse should make. (Select all that apply.) A) Children large enough for a front-facing care seat with a harness for as long as possible. B) Booster seats should be used with both a shoulder and lap belt. C) Children who weigh less than 40 lb should use a rear-facing car seat. D) The back seat remains the safest place for children to ride. E) Many car seats are installed improperly, making them unsafe.

ANSWER A-B-C-D-E- All options are correct and are important safety points for the nurse to make.

Place the steps for using time-out as a disciplinary measure for a 4-year-old in proper order. 1-If the child gets up, replace the child and restart the time. 2-Remove the preschooler to a boring spot. 3-Warn the child there will be a time-out if the behavior does not stop. 4-Parent knows the misbehavior was intentional. 5-Set a timer for no more than 4 minutes.

ANSWER 1) Parent knows the misbehavior was intentional. 2) Warn the child there will be a time-out if the behavior does not stop. 3) Remove the preschooler to a boring spot. 4) Set a timer for no more than 4 minutes. 5) If the child gets up, replace the child and restart the time.

Place these primitive protective reflexes of infancy in the order in which they will disappear as the child matures. A) PLANTAR B) BABINSKI C) STEP D) MORO E) ROOT

ANSWER 1) STEP 2) ROOT 3) MORO 4) PLANTAR 5) BABINSKI The step reflex will disappear at 4 to 8 weeks; the root reflex at 3 months; the Moro reflex at 4 months; the plantar reflex at 9 months; and the Babinski reflex by 12 months.

The nurse is conducting a well-child exam of a 4-year-old child. Which statement would alert the nurse that the child is at risk for iron deficiency? A)"He loves milk and drinks it every time he is thirsty." B) "He enjoys eggs and fortified cereal for breakfast." C) "He does not like spinach, but he does like chicken and beef." D) "He eats a well-balanced diet."

ANSWER A This is likely to result in a very high intake of milk. Excess milk drinking may lead to iron deficiency since the calcium in milk blocks iron absorption. The nurse needs to emphasize this fact and suggest an appropriate daily milk intake. The other statements all include iron-rich foods and would not point to a risk for iron deficiency.

The nurse is teaching the parent of a 5-month-old boy who is concerned about thumb sucking. What should be included in the teaching plan? (Select all that apply.) A)Informing the parent that thumb sucking occurs more often during periods of stress B)Telling the parent this behavior usually decreases by 6 to 9 months of age C)Assuring the parent this behavior won't cause malocclusion D)Advising the parents to draw attention to the issue as this may help child learn to stop E)Advising the parent this behavior is a form of self-comfort

ANSWER A-B-C-E All are accurate and should help the parent better understand and accept the behavior.

The nurse is conducting a physical examination of a 5-month-old infant. Which observation may be cause for concern about the infant's neurologic development? A)The infant grasps a finger when it is placed in the palm. B)The infant displays an asymmetric tonic neck reflex (fencing reflex). C)The toes hyperextend when the bottom of the foot is stroked. D)The anterior fontanel is open and easily palpated.

ANSWER B The tonic neck reflex normally disappears by age 4 months, the palmar grasp reflex by age 6 months, and the Babinski reflex (fanning of toes when sole of foot stroked) by 12 months. Retaining these primitive reflexes may indicate a neurologic abnormality. The anterior fontanel, which remains open for brain growth, closes between 12 and 18 months of age.

A parent is discussing the 10-month-old child with the nurse. Which comment indicates a need for teaching? A)"I wipe my child's teeth every day with a fresh washcloth." B)"My child loves being in the walker and 'zips' around the house." C)"My child gets a few sips of apple juice each day from a regular cup, not a sippy cup." D)"We have safety gates at the top and bottom of our stairs."

ANSWER B Walkers are safety hazards and not recommended by the American Academy of Pediatrics. They cause falls plus promote the ability to reach items on surfaces otherwise inaccessible. The other comments are age appropriate and acceptable practice.

Parents are proud of their toddler's fast-developing fine motor skills. Which skill should the nurse point out as a safety risk? A)Put shapes into matching openings B)Turn book pages C)Ability to turn knobs D)Hold crayon to write

ANSWER C Turning knobs opens doors and may allow the child access to the outdoors or unsafe areas within the home. Close to follow will be the ability to unscrew lids, creating poisoning risks. The other abilities promote learning and involve lesser safety hazards.

A 5-year-old child is being seen at the ambulatory care clinic for a well-child visit. The child is hiding behind the parent. What initial action by the nurse is indicated? The nurse should encourage the child to act like a big kid and stop hiding. Promise to give the child a small toy or sticker if the child will stop hiding. Ask the child's parent to pick the child up and put the child on the examination table. Allow the child to remain "hidden" during the initial part of the interview.

Answer D

While examining a child, the nurse notes quiet, soft sounds each time the stethoscope is moved over the child's chest. The nurse knows that these are not breath sounds. What actions should the nurse take? (Select all that apply.) Auscultate with the child sitting up. Conclude this is a grade 3 heart murmur. Refer the child for further evaluation. Record the location and timing of the sounds. Auscultate with the child lying down.

Rationale: The sounds described are characteristic of a grade 2 heart murmur. The child's heart should be auscultated with the child in two different positions—upright and reclining. Innocent murmurs often disappear when the child's position is changed. Recording the location and timing of the sounds is important to further evaluation and in determining the type and meaning of the murmur. A child with a heart murmur needs further evaluation by an experienced examiner.

A 5-month-old infant being assessed was born at 32 weeks. The nurse doing the well-child check-up should compare the baby to what norms? A)The development of a 10-week-old BThe growth of a 5-month-old C)The development of a 3-month-old D)The growth of a 2-month-old

Answer C The infant was born 8 weeks early. Two months are subtracted from the present age of 5 months. Both growth and development should be assessed using 3-month norms.

PREP U CH 25 LVL 8

PREP U CH 25 LVL 8

For what reasons do nurses work to prevent tobacco use by teens? (Select all that apply.) Smoking is associated with fighting and unprotected sex. Smoking damages the respiratory system. Tobacco use is addicting (nicotine). Teens who smoke are more likely to use alcohol and drugs. Smokeless tobacco damages gums and teeth.

All the options are true and represent reasons nurses work to help teens resist tobacco use.

A mother of a 10-month-old states to the nurse, "I brush my child's teeth every day with flavored kids' toothpaste." Which is the most appropriate response by the nurse? A)"That is great, infants typically hate toothpaste." B)"Toothpaste plays an important role in overall oral health." C)"Drinking water is really all you need to do to rinse your child's mouth." D)"Toothpaste is not necessary; it is the scrubbing that is required."

ANSWER D Toothpaste for infants is not required. The important health technique is the removing of plaque, and that is accomplished through scrubbing of the teeth.

CH 25 newborn/infant

CH 25 newborn/infant

CHAPTER 26 NCLEX

CHAPTER 26 NCLEX

Chapter 29

Chapter 29

Chapter 31

Chapter 31

The healthcare provider has made a notation in the medical record of a 17-year-old adolescent that the client is not demonstrating successful completion of the Erikson stage of development. What behavior would be consistent with this assessment? The adolescent is uncertain and frequently unable to make decisions. The adolescent is distrustful of others. The adolescent is sexually promiscuous. The adolescent is anxious to move away from the parent's home.

ANSWER According to Erickson's stages of development the teen develops a sense of identity. Failure to successfully complete this stage will result in a lack of self-confidence and an inability to see oneself as an independent being. The establishment of the ability to trust is completed in an earlier stage of psychosocial development. A desire to move away from the parental home is not uncommon and is not a sign of impaired navigation of this level of psychosocial development.

A stay-at-home parent wants to purchase commercial toddler meals because the 16-month-old child recently choked on table food. Which food items will the nurse suggest not be given to this child? (Select all that apply.) A)Vegetables such as corn, green beans, and peas B)Hard foods such as nuts, raw carrots, and popcorn C)Fruits such as peaches, pears, and kiwi D)Sticky foods like peanut butter alone, gummy candies, and marshmallows E)Round foods such as hot dogs, whole grapes, and cherry tomatoes

ANSWER To offer soft round foods safely, cut hot dogs in uneven pieces and quarter grapes and cherry tomatoes. This prevents food impacting in an airway. Avoid the hard and sticky foods due to aspiration and airway occlusion risks. The cooked vegetables listed are safe as are the soft fruits.

The nurse is caring for a 5-week-old infant who is spitting up "all the time." This is the mother's first child. What should be the priority nursing intervention? A)Observe the mother while she feeds and burps her infant. B)Offer assurance that spitting up is normal. C)Describe the capacity of a 5-week-old infant's stomach. D)Recommend the mother offer smaller and more frequent feedings.

ANSWER A

Parents and their 35-month-old child have returned to the clinic for a follow-up appointment. Which of the findings may signal a speech delay? A)Uses two-word sentences or phrases B)Talks about a past event C)Asks "why" often D)Half of speech understood by outsider

ANSWER A A child nearly 3 years of age should speak in three- to four-word sentences. The other findings indicate normal expressive language for the age.

The school nurse is preparing health promotion presentations regarding unintentional injuries for a high school health fair. On which topic should the nurse place as the priority when preparing the presentation? A) Motor vehicle safety B) Drug and alcohol use prevention C) Water sports injuries D) Poison prevention

ANSWER A All options should be included in the presentation, but motor vehicle safety has the highest priority because motor vehicle accidents are the leading cause of injury and death followed by poisoning, which includes prescription drug overdose.

The school nurse has just completed a presentation about normal adolescent physical growth to a group of adolescent students. Which comments by the adolescents indicate understanding of the nurse's presentation? (Select all that apply.) A) "Since I just had my first period and I am 14 years old, I probably haven't reached my peak height yet." B) "Our hormones are sure going to cause us to change a lot over our teenage years." C) "It's strange how girls start getting taller before boys start getting taller." D)"Since I'm a 15-year-old guy, I still have a chance to get taller over the next couple of years." E)"It doesn't seem fair that girls typically have more fat than boys."

ANSWER A-B-C-D-E All of these statements demonstrate understanding of normal physical growth during adolescence. The rapid growth during adolescence is secondary only to that of the infant years and is a direct result of the hormonal changes of puberty. Boys' growth spurt occurs later than girls' and usually begins between the ages of 10½ and 16 years and ends sometime between the ages of 13½ and 17½ years. Peak height velocity occurs at approximately 12 years of age in girls or at about 6 to 12 months after menarche. Muscle mass increases in boys and fat deposits increase in girls.

The parent of a 15-year-old child expresses sadness to the nurse that the child is "much more connected to friends than family." What understanding would benefit this parent? (Select all that apply.) A)Adolescents need parental support and guidance as they move toward greater peer involvement. B)Adolescents learn about negotiating differences by spending time with peers. C)Peers can be positive or negative influences. D)Peers provide opportunity to learn and practice social roles. E)Peers are needed for emotional security while stepping away from family.

ANSWER A-B-C-D-E All the statements are true regarding the role peers play in an adolescent's life. The parent may better understand the appeal of the peers.

A parent describes the 4-year-old child's eating habits as "picky" and goes on to describe using, coaxing, and threatening measures to get the child to eat. The nurse determines the child is growing normally. What interventions will the nurse suggest? (Select all that apply.) Focus on quality, not quantity. Offer a variety of healthy foods including those liked and refused. Ignore the pickiness. Approach mealtime matter-of-factly. Allow the child to decide how much the child will eat. Offer only nutritious between-meal snacks.

ANSWER A-B-C-D-E-F Preschoolers often become "picky" eaters but when given the opportunity to include a wider variety in their diet eventually do so. At 5 years of age, children become more receptive to different foods. Too much focus on eating may exacerbate the problem.

The student nurse is assigned to care for a 15-month-old child. When developing a care plan addressing the nutritional needs of the toddler, which client goals demonstrate the student nurse's understanding of these needs? (Select all that apply.) A)"The child will stop drinking from a bottle while in bed within the next 2 weeks." B)"The child will learn to drink from a cup within the next month." C)"The child will be weaned from breastfeeding within the next month." D)"The child will drink amounts suggested by the pediatrician of milk and other fluids on a daily basis." E)"The child will drink milk from a bottle every day for the next 2 months."

ANSWER A-B-D A 15-month-old toddler should be drinking fluid (other than breast milk if still breastfeeding) from a cup because extended bottle feedings at any time of day promote dental caries at this age. Most professional organizations suggest that infants be breastfed for at least 12 months. Breastfeeding beyond 12 months provides nutritional, immunologic, and emotional benefits to the child.

Parents ask for disciplinary guidance for their 4-year-old. The nurse suggests which of these actions? (Select all that apply.) When discussing improper behavior, call the behavior "bad" or "naughty," not the child. Anticipate situations likely to cause misbehavior and redirect the child to another activity. A time-out of about 8 minutes would be appropriate for intentional misbehavior. Books and stories can help preschoolers master proper behavior. Spank with an open hand only and never with an object.

ANSWER A-B-D Spanking is the least effective discipline and discouraged by pediatric professionals. If chosen by parents, it should be infrequent and done only with an open hand. Labeling behavior and not the child supports self-esteem. One minute per year of age is an appropriate length for time-outs. Redirecting from events that tend to lead to misbehavior is wise and reduces conflict. Preschoolers can learn much from stories and books including appropriate behavior.

The parents of a 6-year-old tell the school nurse that they are worried that their child will fall behind other children academically because they are not able to afford expensive toys like computer games and handheld electronic devices. Which are acceptable responses by the nurse? (Select all that apply.) "All of these expensive toys that are advertised and purchased by some people are not necessary for preschoolers. Simple toys like chalk and Legos are great." "Could you possibly save a little money each paycheck in order to be able to purchase such items." "Do you play with your child and provide means of play through things like dolls, puzzles, crayons, and Play-Doh? These are the types of toys suggested for preschoolers." "I understand how this can be frustrating, but rest assured this does not place your child at a disadvantage academically." "There are rental agencies that rent these type of devices. Would that be a possibility financially?"

ANSWER A-C-D Expensive and elaborate toys do not place a child at an academic or developmental advantage. Simple toys that require interactive rather than passive play, and that may include the involvement of the parent, are recommended to foster development.

The parent of a toddler half-jokingly states: "I am so tired of hearing 'NO!' I wish he would stop!" What suggestions will the nurse offer to reduce toddler negativism? (Select all that apply.) A)Avoid ending a request with "Okay?" B)Emphasize what is not to be done: "Don't sit there." C)Avoid "yes" and "no" questions. D)Make statements: "It is time for lunch." E)Offer simple choices: "Blue shirt or red one?" F)Use humor to make "no" funny: "Do cows bark?"

ANSWER A-C-D-E-F All suggestions except what not to do are positive and reduce the opportunity to say "no."

The nurse realizes that the 5-year-old's growth chart and BMI indicate the child is at risk for obesity. What other findings reinforce this risk? (Select all that apply.) A) Food used as reward or punishment B) Fast food eaten once a month C) Television on during meals D) Mealtimes organized and regular E) A parent who is overweight F) Expectations to eat everything on plate

ANSWER A-C-E-F Overweight parents, food used as reward or punishment, TV viewing during meals, and the "clean plate club" all are factors that contribute to weight gain. The frequency the family eats fast food is not excessive. Regular, structured meals result in less snacking and more nutritious meals. Family eating together is bonding.

The nurse is reviewing sleep and rest activities of a 16-month-old child with the parents. The parent states, "I have told my spouse it is unhealthy for our child to sleep with us. It's time for the child to sleep in one's own bed. What do you think?" What is the nurse's best initial response? A)"I will note this in your child's chart for the healthcare provider to see." B)"It must be difficult for the two of you to both feel strongly about what is best for you and your child." C)"Co-sleeping is viewed acceptable by some professionals, but interfering with the child's independence by others." D)"Have you thought about just trying to let your child sleep in one's own bed to see how it goes?"

ANSWER B Acknowledging the struggle between the mother and father allows for open conversation about the sleeping arrangement, which may lead to an acceptable resolution. Stating the views of professionals about cosleeping, while is accurate, does not address the parent's voiced concerns, nor does noting the sleep activities in the chart. Suggesting the child sleep in one's own bed is not up to the nurse to suggest, and it does not address the issue.

The parents of a 5-year-old call the nurse for advice about night terrors. The child has had them nightly for almost 2 weeks. What is the most appropriate intervention? Tell the parents to be sure to wake the child up during the next episode. Wake the child up nightly 30 to 45 minutes after going to sleep. Encourage the parents to let the child fall asleep in a safe place such as their bed. Explain that this is a developmental phase that will not last.

ANSWER B Awakening children early in their sleep cycle often interrupts the night terror events and should be continued nightly for about 7 days. Suggesting the parent wake the child during a night terror is both difficult and ineffective. A major reason parents find night terrors frightening is that the child does not respond to the parent's presence. Indicating this is a developmental phase may be somewhat accurate, but does not assist the parents in handling the situation. Having the child fall asleep in a "safe" place or the parents' bed is likely to predispose to other sleep problems without helping to solve this one.

A 12-month-old seen at a walk-in clinic weighed 8 lb 4 oz (3.75 kg) at birth. Weight now is 20 lb 8 oz (9.3 kg). The nurse determines: A)The weight assessment is blatantly inaccurate. B)The child weighs less than expected for age. C)The child weighs the expected amount for age. D)The child weighs more than expected for age.

ANSWER B Birth weight should triple by 12 months. The child should weigh near 24 lb 12 oz (11.25 kg). The child is underweight for age.

While evaluating the development of a 10-month-old boy, a nurse hides the boy's stuffed animal behind her back. The boy crawls around the examination table to look behind the nurse's back for the stuffed animal. Which developmental phenomena has this infant demonstrated? A)Depth perception B)Object permanence C)Hand regard D)Binocular vision

ANSWER B By 10 months, an infant looks under a towel or around a corner for a concealed object (beginning of object permanence, or become aware an object out of sight still exists). Hand regard, which is typically demonstrated by 3-month-olds, is a phenomenon that involves the infant holding his hands in front of his face and studying them. Binocular vision, which is the ability to fuse two images into one, is demonstrated by 2-month-olds when they follow moving objects with their eyes. Depth perception allows 7-month-olds to transfer toys from hand to hand.

The nurse is examining a 2-year-old child for speech and language development. Which finding would suggest a delay in speech development? A)The child repeats what the parents say out of context and at random moments. B)The child does not use the names of familiar objects. C)The child puts together sentences of two words. D)The child does not speak clearly but shows understanding of what is said.

ANSWER B By 24 months most children will name objects familiar to them in their daily lives. Not doing so is strong evidence that a speech delay may exist. Repeating words heard or phrases out of context (echolalia) is normal and a way to practice words and incorporate them in the vocabulary. At 2 years, most children understand much more than they can clearly repeat. Using two-word sentences is a developmental expectation at this age.

A 17-year-old child needs to increase iron intake but is concerned about weight gain. The nurse will suggest which foods? A)Egg white omelet, broccoli, pear B)Chicken, whole-wheat bread, watermelon C)Cottage cheese, crackers, oranges D)Raspberry yogurt, granola, apple

ANSWER B Chicken, whole-wheat bread, and watermelon are all foods high in iron. The calorie content will not promote weight gain. All the other options do not include good sources of iron but are nutritious foods. Calorie content is also acceptable.

A staff nurse is talking about Piaget's theory with a nursing student. Infants are in the sensorimotor stage of cognitive development during which object permanence is mastered. An example of an infant displaying this ability is: A)smiling at oneself in the mirror. B)looking for a toy in the crib at the last place the infant saw it. C)pushing a spoon from the high chair tray to the floor. D)shaking a rattle to enjoy the sound.

ANSWER B Looking for the toy where it was last seen indicates object permanence. The infant knows the object still exists even though the infant cannot see it. All the rest of the infant's behaviors show use of the senses and motor activity but do not illustrate object permanence.

What feeding practice used by the parents of an 8-month-old should the nurse discourage? A)Continuing to offer foods the child rejects B)Placing all liquids given the child in a "no spill" sippy cup C)Giving the child soft table food and finger foods Including the infant at family meals in the high chair

ANSWER B No spill sippy cups promote sucking rather than teach cup drinking. Reserve these for times avoiding spills is essential. The other feeding practices are age appropriate and safe. Soft table and finger foods promote accepting new textures and self-feeding. Reoffering rejected food allows the child to accept it when ready. Including the infant at the family table provides for modeling of eating behaviors and socialization.

The nurse is providing teaching about good nondairy sources of calcium for preschoolers. Which of these fruits contains the most calcium? A) Apple B) Orange C) Banana D) Peach

ANSWER B Rationale:A medium orange contains 50 mg of calcium and is a good nondairy choice. The other fruits are healthy choices but do not contain as much calcium.

The nurse is examining the heart and peripheral perfusion of an 8-year-old. The nurse will assess the apical impulse at which location? The fourth intercostal space just medial to the child's left midclavicular line The fourth intercostal space at the left midclavicular line The fifth intercostal space medial to the left midclavicular line The fifth intercostal space lateral to the left midclavicular line

Answer D

The nurse is educating the parents of a newborn prior to discharge home. The parents demonstrate teaching was successful when making which statement(s)? Select all that apply. A)"We should get some rest in about 1 month when the newborn starts sleeping through the night." B)"I understand it is normal for newborns to lose 5% to 10% of their bodyweight after birth." C)"My newborn can see up-close things, like our faces, better than things at a distance." D)"I plan to add a little rice cereal to my breast milk so my newborn will sleep longer at night." E)"I will not be concerned if my newborn has stools that begin to have a yellowish color to them."

ANSWER B-C-E Newborn stools will become yellowish in color after the first few days of life. Newborns typically lose 5% to 10% of their birthweight the first few days of life, and begin to gain weight after this period. Newborns have better up-close vision and begin to recognize human faces during their newborn stage. Most infants will not sleep through the night until about 3 months of age. There is no evidence that rice cereal keeps a newborn from waking and the practice of feeding rice cereal to newborns is discouraged by physicians as the newborn needs formula or breast milk specifically.

A parent reports her 2-year-old daughter has gradually eaten more and more poorly since her 1-year-old well-child visit. The nurse assesses the child's growth and development as normal. What concept explains the parent's concern? A)Poor role modeling B)Improper snacking C)Physiologic anorexia D)Iron-deficiency anemia

ANSWER C Physiologic anorexia results when the rapid growth of infancy slows in toddlerhood and less food is required for body size. Sugary and fatty snacks as well as role modeling poor eating habits can influence a toddler's food intake negatively. However, this is not what the parent is describing. Iron-deficiency anemia is extremely common among toddlers and could be present in this child, but this is not the reason for her lessened food intake.

Parents of a 3½-year-old indicate they spend time with grandparents who live near a lake. The nurse will emphasize: enrolling the child in swimming lessons. using and renewing sunscreen regularly. having the child wear a personal flotation device whenever near or on the water. adults learning infant/child cardiopulmonary resuscitation (CPR).

ANSWER C Preschool children are safe around water only when adult supervision is constant. Wearing a personal flotation device adds additional protection and should be as routine as "buckling up" in the car. At 4 to 5 years of age, most preschoolers are mature enough to become swimmers; yet knowing how to swim does not make the preschooler safe without supervision. CPR is a life-saving skill and using sunscreen will protect the skin, but neither will be a factor in preventing drowning.

The nurse is performing an assessment on a 8-month-old infant. The infant's medical history notes that he was born at 32 weeks' gestation. The infant is progressing normally. At what adjusted age should the nurse expect the infant's developmental accomplishments? A)By 8 months of age, the child's skill level will vary greatly and cannot be predicted. B)The infant can be expected to display developmental skills consistent with a 8-month-old infant. C)The infant will most likely resent with developmental skills consistent with a 6-month-old infant. D)The infant will likely show the skills of an infant with the adjusted age of 7 month.

ANSWER C When assessing the growth and development of a premature infant, the nurse will use the infant's adjusted age to determine expected outcomes. To determine adjusted age, the nurse subtracts the number of weeks that the infant was premature from the infant's chronologic age. The infant who was born at 32 weeks' gestation was 8 weeks or 2 months premature. To determine the adjusted age, the nurse subtracts 2 months from the chronologic age of 8 months: 6 months.

The mother of a 6-week-old infant reports she doesn't know if her child recognizes her face yet. What response by the nurse is most appropriate? A)"Recognition of faces and voices will come with time." B)"Recognition of this type begins around 8 weeks of age." C)"Since about 4 weeks of age your child has been able to recognize those who are around him often." D)"Don't worry. He knows you are his mother."

ANSWER C At 1 month of age the infant can recognize by sight the people he or she knows best. Telling the child's mother that this will come with time is not correct as this developmental milestone has already occurred. Telling her not to worry minimizes her questions and concerns.

The nurse is preparing a list of abilities of 10-month-olds to use in teaching a parenting group. Which ability should appear at this age? A)Uses two or three words with meaning B)Sits from standing position C)Feeds self with spoon (but spills) D)Cruises around furniture

ANSWER C At 10 months, this ability appears and is practiced often in preparation for later independent walking. All the rest of the skills take an additional 2 months to develop and appear around age 1 year.

A nurse is talking to and making facial expressions at a 9-month-old baby girl during a routine office visit. What is the most advanced milestone of language development that the nurse should expect to see in this child? A)The infant imitates her father's cough B)The infant squeals with pleasure C)The infant says "da-da" when looking at her father D)The infant coos, babbles, and gurgles

ANSWER C By 9 months, an infant usually speaks a first word: "da-da" or "ba-ba." The other answers refer to earlier milestones in language development. In response to a nodding, smiling face, or a friendly tone of voice, a 3-month-old infant will squeal with pleasure or laugh out loud. By 4 months, infants are very "talkative," cooing, babbling, and gurgling when spoken to. At 6 months, infants learn the art of imitating. They may imitate a parent's cough, for example, or say "Oh!" as a way of attracting attention.

Parents say they have been using measures to lessen the struggle of getting their preschooler to bed at night and to sleep. Which practice will the nurse suggest they discontinue? A) Providing a nightlight B) Taking the TV set out of the child's room C) Allowing the preschooler to fall asleep wherever and whenever the child is tired enough D) Eliminating caffeine sources beginning late afternoon

ANSWER C Consistent bedtimes and places for sleep promote good sleep habits. Caffeine (soft drinks) interferes with sleep. A nightlight can reduce fear of the dark common in preschoolers. Removing the TV from the child's room prevents viewing and screen light from keeping the child awake.

The nurse is conducting a skin assessment of a newborn. The examination reveals a light pink macule on the back of the neck. The nurse understands that this is a normal variation and is most likely which type of birthmark? Nevus flammeus Petechiae Salmon nevus Purpura

Rationale: A light pink macule on the back of the neck is a salmon nevus or "stork bite." A nevus flammeus (port wine stain) is dark purple-red. It is a flat patch that grows with the child. Petechiae are pinpoint reddish macules that do not blanch when pressed. Purpura are large purple macules created by bleeding under the skin.

The nurse is providing a nutrition workshop for the parents of infants. The nurse understands that further instruction is required when hearing which comments from the parents? Select all that apply. A)"I plan on encouraging my son to cook with me when he is old enough so that he will enjoy a variety of foods and learn how to cook too." B)"I try to eat healthy in front of my daughter so she will hopefully pick up good eating habits." C)"I let my child eat whatever he wants right now so that we don't argue about food. Hopefully he will like healthy foods when he grows up." D)"I have tried at least 10 times with every green vegetable and I can't get my son to like them." E)"Food is so expensive. I can't afford for my child to leave any food on the plate."

ANSWER C-D-E Encouraging children to eat everything on their plate can lead to overeating and obesity. Children may need to be exposed to new food at least 20 times before determining if they like it or not. Letting a child eat whatever he wants does not lead to good choices as the child matures.

The parent of a preschooler reports that the child seems to believe in magic. The parent voices concern that this "fantasy world" may become a problem. What response by the nurse is indicated? (Select all that apply.) A)"This type of imagination is not normally seen until a child is school aged." B)"While imagination is normal, this type of fantasy world can cause problems for your child and should be discouraged." C)"This type of thought process allows your child to begin to observe the differences in the world." D)"Fantasy play is most often seen in lonely children in an attempt to occupy themselves." E)"Your child is engaging in what we call magical thinking."

ANSWER C-E Magical thinking is a normal part of preschool development. In magical thinking, the preschooler believes that his or her thoughts are all-powerful. The fantasy experienced through magical thinking allows the preschooler to make room in his or her world for the actual or the real. Through make-believe and magical thinking, preschool-age children satisfy their curiosity about differences in the world around them. There is nothing problematic about this type of imagination.

What action shows an example of Erik Erikson's developmental task for the infant? A)The infant plays the game peek-a-boo. b)The infant smiles as people walk past the crib. C)The infant cries when they have a wet diaper. d)THe infant cries and the caregiver picks the child up.

ANSWER D

The nurse is providing client education to the parent about bathing the infant. What would be important to instruct the parent? A)Never use soap on an infant's hair B)Soap lubricates and oils an infant's skin C)Infants need a daily bath D)Bath time provides an opportunity for play

ANSWER D The work of children is play. Play provides a natural way for the infant to learn. In early infancy, infants prefer their parents rather than toys. Parents can talk and sing to infants during feeding, bathing, and changing diapers. Infants do not need a daily bath as long as the diaper area is washed with diaper changes. Soap is actually drying to an infant's skin. Washing the hair with soap can help remove excess oil.

The nurse is teaching parents how to avoid a power struggle with their 2-year-old child. Which comment indicates that more teaching is needed? A)"We will give our child a choice whenever possible." B)"Childproofing our home will make it less necessary to say 'No!'" C)"Both of us, as parents, will agree on and consistently enforce the limits we set." D)"We will make sure our child shares toys with cousins of this age."

ANSWER D A 2-year-old's thinking is not mature enough to include the concept of "sharing." The child sees situations from a child's point of view (egocentrism). That makes sharing with another beyond what the child can accept as an understandable action. All the other statements are age appropriate and support the capabilities and growing independence of the toddler.

The nurse is recommending food items for an 18-month-old child. Which ones will benefit the child's neurologic system most? A)Oatmeal pancakes with bananas B)Vegetable soup, whole wheat bread, and blueberries C)Ground beef, broccoli, and apple slices D)Peanut butter on crackers, cheese, and whole milk

ANSWER D At 18 months, the child's neurologic system is still developing rapidly and needs dietary fat for myelinization. Peanut butter on crackers, cheese, and whole milk all provide this. The other foods are appropriate for the toddler as part of a balanced diet.

The nurse is doing a well-child exam of a 5½-year-old child. Which statement by the parent would alert the nurse that further teaching is needed about healthy eating habits? A)"We eat fast food less than once a month." B)"I try to set a good example and eat a variety of fruits and vegetables." C)"My child enjoys planning meals and helping me in the kitchen." D) "Our family rule is that all children clean up their plates at each meal, and we feed them a lot."

ANSWER D Children should not be expected to always eat everything on their plates, nor should they be rewarded for doing so. Appropriate serving sizes are important here. Children need to learn to self-regulate and eat only until full. The other statements indicate knowledge of healthy eating habits and are useful in developing positive attitudes toward eating and toward trying different foods.

What action shows an example of Erik Erikson's developmental task for the infant? A)The infant cries when they have a wet diaper. B)The infant plays the game peek-a-boo. C)The infant smiles as people walk past the crib. D)The infant cries and the caregiver picks the child up.

ANSWER D Erikson's psychosocial developmental task for the infant is to develop a sense of trust. The development of trust occurs when the infant has a need and that need is met consistently. Crying with a wet diaper without a change of the diaper leads to an unmet need. Playing peek-a boo and smiling are developmental tasks that indicate a normal healthy, happy baby. These would be attributed to Piaget theory.

What statement by the parent of a 20-month-old indicates a need for further teaching about nutrition? A)When my child doesn't eat well at meals we give nutritious snacks." B)"New foods are offered along with ones my child likes." C)"My child drinks three 6-ounce cups of whole milk each day." D)"I give my child juice at breakfast and when my child is thirsty during the day."

ANSWER D High juice intake can contribute to either obesity or appetite suppression. None is needed, but if juice is given limit the amount to 4 to 6 ounces daily. Water should be the choice for thirst. The other statements support good toddler nutrition. Whole milk is needed through age 2 years. Two cups daily is adequate. Nutritious snacks support quality intake when quantity is poor. New foods offered with old ones provide sameness along with the new.

A client who is breastfeeding asks the nurse if she can give the newborn a pacifier. Which nursing response is most appropriate? A)"I will request the lactation consultant come talk to you about pacifier usage while breastfeeding." B)"This decision should be made by you and your partner based on your personal preferences." C)"I know a lot of people who breastfed and also gave their newborns a pacifier." D)"It is recommended to wait until breastfeeding is well-established before introducing a pacifier."

ANSWER D It is recommended to wait to introduce a pacifier once breastfeeding is well-established, which can take about 1 month. This is to limit nipple confusion and promote an adequate milk supply. Stating other people have done this does not provide education to the client, nor does it address this specific client's situation. While the decision is up to the newborn's parents, this response does not address the client's concern. Requesting a lactation consultant come does not address the client at this moment. The nurse can provide education now, and also request the consultant for follow-up information.

Parents tell the nurse their 3½-year-old refuses to eat meat but are pleased the child drinks "lots of milk." What risk does the nurse identify? A) Obesity B) Interference with growth C) Dental caries D) Iron deficiency

ANSWER D Meat is an important iron source while calcium in milk consumed in large quantities can block iron absorption. Alternate protein sources can replace the meat in the child's diet for growth. Excess milk intake that boosts calories consumed can be an obesity-causing factor. Lactose from milk in constant contact with the teeth can promote development of dental caries. However, these risks are slight, with the iron deficiency risk pronounced.

The parent of a Black adolescent voices concern to the nurse because the child, "has gotten her period before all of her friends." How should the nurse respond? A)"How old are most of her friends? Maybe that's the issue instead of it being a sign of something abnormal." B)"Some girls just get their period earlier than others." C)"I will be sure to let the health care provider know this. We don't want to miss something that may be wrong." D)"That must be difficult, but on average Black girls start their period earlier than other ethnicities."

ANSWER D Menarche, the first menstrual period, usually begins between the ages of 9 and 15 years (average 12.8 years), but on average Black girls reach menarche earlier than girls of other ethnic groups. This response addresses the parent's concern. The other responses do not address the parent's concern or may lead the parent to think this is an abnormal occurrence.

The nurse is assessing speech development in the 2-year-old whose family uses both Spanish and English in the home. What finding is of concern? A)The parents understand the child much of the time. B)Some words the toddler speaks are a blend of English and Spanish. C)The child mixes words from the two languages within a sentence. D)The toddler speaks 15 words between the two languages.

ANSWER D Of concern is speaking only 15 words between Spanish and English. At 20 months, the bilingual child should use 20 words. The other findings fit the norms for a bilingual child.

The nurse is visiting a day care center and watches two toddlers at play. What best describes the play observed? A)Sharing stuffed animals B)Playing for significant stretches of time in one activity C)Sitting quietly with several toys D)Playing alongside one another

ANSWER D Playing beside one another is parallel play and typical of toddlerhood. Toddlers have short attention spans and frequently change toys and activities. Because toddlers are egocentric, sharing is difficult. Toddlers move about and explore during play rather than sit quietly.

With summer approaching, the nurse discusses sun safety with a group of adolescents. Which suggestion is most likely to be followed? A)Limit sun exposure between 10:00 AM and 2:00 PM. B)Wear a hat when in the sun. C)Do not attempt to get a tan. D)Wear sunglasses.

ANSWER D Sunglasses can be a fashion statement and are likely to be accepted. The other pieces of advice are less appealing to adolescents who link a tan body to beauty. Unfortunately sun exposure is cumulative over a lifetime, and much of it occurs in youth, making protecting young skin important.

The parents of a 4-day-old infant report concern about the infant's weight loss. What is the best response by the nurse? A)"Weight loss after birth is normal." B)"Babies will begin to rapidly regain weight and will double birth weight around 6 months of age." C)"Babies may lose up to 10% of their body weight in the first month of life." D)"With appropriate nutrition weight gain will commence with a return to the birth weight within 2 weeks."

ANSWER D The average newborn weighs 7 lb 8 oz (3,400 g) at birth. Newborns lose up to 10% of their body weight over the first week of life. The average newborn then gains about 30 g per day and regains his or her birthweight by 10 to 14 days of age. Most infants double their birth weight by 4 months of age and triple their birth weight by the time they are 1 year old.

A teacher refers a student to the school nurse because the student is frequently falling asleep during class. After talking with the student, the nurse is most concerned by which statement by the student? A)"I just can't seem to stay awake during that class because it's boring." B)"My mom keeps telling me to turn off my television when I go to bed." C)"I guess I need to be more careful about my curfew on school nights." D)"I get 7 hours of sleep every night so I don't know why I am so tired."

ANSWER D The average number of hours of sleep that teens require per night is 8.5 to 9.5 due to rapid growth that occurs during these years. Following a curfew and limiting distractions at bedtime can help provide the student with adequate hours of sleep each night.

The parents of a 5-year-old child are discussing bicycle safety with the nurse. What comment indicates further teaching is needed? A)"Our son always wears a helmet." B)"He never rides in the street." C)"He is able to ride without training wheels." D)"We just got him a new bike he can grow into."

ANSWER D The bicycle should fit the rider. The balls of his feet should reach both pedals while he is sitting on the seat and has both hands on the handlebars. The other comments describe no safety risk.

The nurse is assessing the oral cavity of a 6-month-old child. When palpating the location in which the first primary teeth erupt, which location is being assessed? A)upper lateral gumline B)lower lateral gumline C)upper central gumline D)lower central gumline

ANSWER D The lower central incisors are usually the first to appear, followed by the upper central incisors.

The parents of a 2-year-old child report to the nurse because their child is "such a picky eater." Which recommendation would be most helpful for developing healthy eating habits in this child? A)Advising the parents to minimize distractions at mealtime B)Assuring the parents that food jags are normal, and they can be honored safely C)Encouraging the parents to eat a variety of wholesome foods themselves D)Offering a variety of foods along with the foods the child likes

ANSWER D Toddlers require fewer calories proportionately than infants, and their appetite decreases (physiologic anorexia). Offering a variety of healthy foods along with foods the child likes will acknowledge preferences while keeping the door open to new foods. Prolonged particular food preferences (food jags) are common. It is also important that mealtime be calm, pleasant, and focused on eating. Toddlers mimic behaviors observed. It is important that parents set a good example with their mealtime behaviors and food choices. All options encourage the development of healthy eating habits, but at this time, variety plus preferred foods will be most helpful.

Which suggestion by the nurse meant to promote good dental health in the 15-month-old is inappropriate? A)Wean the child from the bottle. B)Avoid grazing (continual snacking) throughout the day. C)Arrange for your child's first dental visit as soon as possible. D)Brush your child's teeth with a pea-sized amount of fluoride-containing toothpaste.

ANSWER D Using fluoride toothpaste prior to age 2 years promotes development of fluorosis. The first dental visit should be made at 1 year. This check-up is overdue. Continual snacking and bottle drinking keep the teeth in contact with cariogenic substances for extended periods.

The nurse is caring for an adolescent athlete who is being seen for a fractured arm. The adolescent's parent reports that this is the third sports injury in the past 2 years. The parent asks the nurse why the child who seems so healthy seems to continue to have injuries. What information should be included in the nurse's response? A)Some children are accident prone. B)These are accidents and random in occurrence. C)There may be some underlying problems that the child should be evaluated for. D)The bones, joints and tendons of teens are vulnerable to injury due to their rapid state of growth.

ANSWER D Rapidly growing bones, muscles, joints, and tendons are more vulnerable to unusual strains and fractures. While some people may seem to be accident prone, this is most likely the result of the stage of physical growth experienced during adolescence. There is no evidence the child has any underlying medical conditions.

The nurse is providing anticipatory guidance regarding the respiratory development of a 4-week-old infant for the parent. Which action is accurate? A)Telling the parent that abdominal breathing disappears by 9 month of age B)Informing the parent that the respiratory system reaches maturity similar to the adult's by 12 months of age. C)Advising the parent that the infant's usual respiratory rate should slow to about 20 breaths per minute by age 6 months D)Explaining to the parent the risk for infection is high due to the lack of antibodies

ANSWER D Rationale:Attributing frequent infections to a lack of antibodies is accurate. The infant lacks IgA in the mucosal lining of the upper respiratory tract. The infant's respiratory rate drops to 20 to 30 breaths per minute by the end of the first year. Abdominal breathing persists until 6 to 12 years of age. The respiratory system matures by age 7 years.

A parent mentioned to the nurse that the usually smiling, happy 8-month-old child was clingy and intensely serious when the grandparent visited from a distant city. The nurse explained the child was experiencing: A)separation anxiety. B)colic. C)cephalocaudal development. D)changes in temperament. E)stranger anxiety.

ANSWER E Stranger anxiety occurs around 8 months and manifests as the parent described. This behavior indicates the infant sees himself as a separate person. The other options are incorrect and not related to social/emotional development.

The nurse is helping the parent of a 5-month-old infant understand the importance of developmentally appropriate play. Which one of the toys best meets the needs of this child? A)A yellow rubber duck for the bath B)Brightly colored stacking toy C)Pots and pans from the kitchen cupboard D)A push-pull toy

ANSWER a The rubber duck is most appropriate. It is safe, visually stimulating while bobbing on the water, and adds pleasure to bath time. A push-pull toy promotes skill for a walking infant. Pots and pans from the kitchen cupboard are played with successfully after sitting is mastered. A 5-month-old does not have the fine motor coordination to use stacking toys.

A 17-year-old adolescent on the high school swim team tells the nurse diet changes during swim season that involve cutting the carbohydrates in the diet to 30% to improve swim times. What responses by the nurse are appropriate? (Select all that apply.) "As long as you are taking in around 2,500 calories per day you should be just fine." "Twenty-five to thirty-five percent of your diet should be carbohydrates so you fall into the acceptable range." "Can you tell me the reason you feel the need to cut your carbohydrates when your activity level is high?" "Cutting carbohydrates is a good way to keep your weight down during swim season." "Since you are so active, your carbohydrate intake should comprise 45% to 65% of your daily diet."

Answer Rationale: Adolescents who are moderately active require between 2,200 and 2,800 calories per day and 45% to 65% should come from carbohydrates. Carbohydrates should not be cut, especially during an athletic season when energy use is increased. Asking the student why he or she are cutting carbohydrates is appropriate in order to help the nurse address the issue effectively.

The nurse is collecting data from an adolescent being seen for a well-child check-up. During the interview, the adolescent reports sleeping about 6 hours per night during the week but is able to sleep 8 to 9 hours per night on the weekend. What response by the nurse is most appropriate? "Let's talk about ways to increase the amount of rest you get during the week." "I am glad to see you are getting the amount of rest you need." "You would benefit by trying to sleep a little less on the weekends." "Sleeping during the weekend to catch up on missed rest is a good thing to do."

Answer Rationale: It is recommended that adolescents get 8 to 9.5 hours of sleep per night. Adolescents who do not get adequate rest during the week often sleep more hours on the weekend. The best practice would be to determine ways to improve daily rest patterns.

The parent of a 14-year-old girl voices concerns that the child is likely to be not getting enough nutrients. How many calories should be ingested by the child daily? The range for caloric intake for an adolescent girl is 1,700--2,000 per day. The adolescent girl who is active should consume about 2,000 calories per day. At least 1,800 calories per day is recommended for an adolescent girl. The adolescent should not consume more than 2,500 calories per day.

Answer Rationale: Adolescent girls who are moderately active require about 2,000 calories per day.

The school nurse is preparing health promotion presentations regarding unintentional injuries for a high school health fair. On which topic should the nurse place as the priority when preparing the presentation? Motor vehicle safety Water sports injuries Poison prevention Drug and alcohol use prevention

Answer Rationale: All options should be included in the presentation, but motor vehicle safety has the highest priority because motor vehicle accidents are the leading cause of injury and death followed by poisoning, which includes prescription drug overdose.

What anticipatory guidance can the nurse provide the girl who has noted the development of breast buds? The growth spurt will begin immediately with menarche. Menarche should follow in about 2 years. Breast development will be complete with 12 months. Adult height will be reached at the time of menarche.

Answer Rationale: Menarche usually follows within 2 years of the first signs of breast development. Peak height velocity (PVH) in girls occurs 6 to 12 months following menarche. It does not follow immediately. Breast development progresses through several stages and will not be complete until late puberty. Adult height is not reached at the time of menarche but about 6 to 12 months following menarche.

A 15-year-old adolescent tells the nurse about a desire to get a tattoo. What response by the nurse is most appropriate? "You are too young to have a tattoo." "Tattoos are invasive and there is the potential for disease with their application." "If you are determined to get a tattoo you need to get a small one so you can hide it if you want." "You need to remember that a tattoo will be with you forever."

Answer Rationale: Piercings and tattoos are becoming increasingly common in teens. It is important to ensure that the teen and family understand the potential risk for disease with the application of the tattoo. The teen is young, but that does not change the fact he wants to get a tattoo. Reminding the teen that it is a permanent alteration in the skin is important but not the most important response by the nurse.

The nurse is auscultating the heart of a 6-month-old child. Which finding would warrant further investigation? Incorrect option A split S2 at the apex Unanswered option Heart rate of 120 Correct option S1 varies in intensity. Unanswered option Variation in heart rate durin

Answer Rationale: The S1 should not vary in intensity at a particular point. If it does, this may indicate a cardiac arrhythmia, and the child should be referred for further evaluation. A split S2 at the apex occurs in many infants and young children. The other findings are within the normal range for a child of 6 months.

The nurse is providing anticipatory guidance regarding the respiratory development of a 4-week-old infant for the parent. Which action is accurate? A)Informing the parent that the respiratory system reaches maturity similar to the adult's by 12 months of age. B)Explaining to the parent the risk for infection is high due to the lack of antibodies C)Telling the parent that abdominal breathing disappears by 9 month of age D)Advising the parent that the infant's usual respiratory rate should slow to about 20 breaths per minute by age 6 months

Answer B Rationale: Attributing frequent infections to a lack of antibodies is accurate. The infant lacks IgA in the mucosal lining of the upper respiratory tract. The infant's respiratory rate drops to 20 to 30 breaths per minute by the end of the first year. Abdominal breathing persists until 6 to 12 years of age. The respiratory system matures by age 7 years.

Parents state they are "worn out" at their child's 6-month check-up because their child awakens each night and cries. The nurse suggests which measures? A)At bedtime, rock the child to sleep and then place in crib. B)Add rice cereal to the evening bottle to prevent hunger and awakening. C)Establish a quieting ritual before bed. D)During night awakening, do not interact with the child.

Answer C Bedtime rituals help both promote sleep. During awakenings, the parents should interact with the child, but minimize attention and stimulation. Adding rice cereal to bottles does not promote sleeping through the night and isn't recommended. Rocking the child to sleep and then placing them into the crib does not teach the child to self-soothe and fall asleep independently.

Chapter 32

Chapter 32

The nurse working with children and families knows there are certain universal screening tests all children should receive. Which tests are included in this group? (Select all that apply.) Denver II Lead levels Hyperlipidemia Phenylketonuria Auditory brain stem response

D-E

The nurse is assessing deep tendon reflexes on a child admitted for severe dehydration. The assessment reveals hyperactive reflexes. How should the nurse document this finding? 2+ 3+ 4+ 1+

Rationale: Deep tendon reflexes are graded by the strength of the response using the standard scale from 0 to 4+: 0, no response; 1+, diminished or sluggish; 2+, average; 3+, brisker than average; 4+, very brisk, may involve clonus.

The nurse is visualizing the ear canal and tympanic membrane of a 14-month-old client. Which finding would warrant further investigation? A mobile tympanic membrane A gray tympanic membrane A bubble behind the tympanic membrane A pearly pink membrane

Rationale: A bubble behind the tympanic membrane is not a normal finding and indicates a need for further investigation. The other findings are within normal limits.

During a health maintenance visit, the nurse determines that a 12-year-old client is overweight for his age and height. Which approach for promoting healthy weight will help the client maintain self-esteem? Showing the client where he or she plots on a growth chart Suggesting the client record one's weight weekly on a calendar Encouraging the family to purchase a scale Emphasizing how activity and a nutritious diet promote health

Rationale: A health-centered rather than a weight-centered approach places the emphasis on health and is best for supporting self-esteem. A combination of diet and exercise is the recommended way to control weight. Showing the client where he or she plots on a growth chart, purchasing a scale, and weekly recording of weight all focus on numbers and not on an overall healthy lifestyle.

The school nurse has just completed a presentation about normal adolescent physical growth to a group of adolescent students. Which comments by the adolescents indicate understanding of the nurse's presentation? (Select all that apply.) "It's strange how girls start getting taller before boys start getting taller." "Since I just had my first period and I am 14 years old, I probably haven't reached my peak height yet." "Our hormones are sure going to cause us to change a lot over our teenage years." "It doesn't seem fair that girls typically have more fat than boys." "Since I'm a 15-year-old guy, I still have a chance to get taller over the next couple of years."

Rationale: All of these statements demonstrate understanding of normal physical growth during adolescence. The rapid growth during adolescence is secondary only to that of the infant years and is a direct result of the hormonal changes of puberty. Boys' growth spurt occurs later than girls' and usually begins between the ages of 10½ and 16 years and ends sometime between the ages of 13½ and 17½ years. Peak height velocity occurs at approximately 12 years of age in girls or at about 6 to 12 months after menarche. Muscle mass increases in boys and fat deposits increase in girls.

The nurse practitioner inspects a toddler's teeth. The nurse practitioner encourages the family to establish a dental home in order to achieve optimal dental health for the various family members. Which does the nurse recognize as the rationale(s) for the establishment of a dental home? (Select all that apply.) Poor oral health care for children can result in systemic health problems. Certain dental interventions including fluoride treatments could significantly reduce the cost of oral care for children. Comprehensive health care is possible only if oral health is part of the equation. Dental care is the most unmet health need of children in the United States. A dental home with secure comprehensive care from infancy to adolescence

Rationale: All statements are true and underscore the importance of good oral health care best achieved through a dental home.

The parents of an 8-year-old state, "I am happy that our child is healthy," when the nurse says that the child falls into the 95th percentile for BMI. How should the nurse respond? "I will let the healthcare provider know that your child is in the 95th percentile for BMI." "For a child BMI between the 85th and 95th percentile alerts us to the risk for being overweight. Let's talk about your child's diet and activity level." "Being in the 95th percentile for BMI is not a good thing. Your child is on the verge of obesity. It would be a good idea to consider this with meal planning." "The 95th percentile is not an indication of health."

Rationale: BMI between the 85th and 95th percentiles for children between the ages of 2 and 20 indicates risk for overweight. BMI greater than the 95th percentile indicates the child is overweight. Informing of the parents of these findings and discussing diet and activity effectively address the issue in a therapeutic way.

For reasons of anticipatory guidance, nurses should be aware that menarche appears earlier in some ethnic groups than others. In which ethnic group is menarche likely to appear first? Unanswered option White Americans Unanswered option Indian Americans Unanswered option Mexican Americans Correct option Black Americans

Rationale: Black girls on average reach menarche slightly earlier than White, Mexican American, and Indian American girls.

The parent of a 4-day-old infant brings the child to the clinic concerned about the infant's hands and feet appearing blue. What is the nurse's best response during the admission process? "This is often a normal occurrence in infants of this age but we will have the healthcare provider assess your baby." "It is good that you brought your infant in when you noticed this discoloration in the hands and feet." "This is likely acrocyanosis. I will document this in the medical records." "This is commonly occurs as a result from an immature circulatory system completing the switch from fetal to life after birth."

Rationale: Blueness of the hands and feet, known as acrocyanosis, is normal in babies up to several days of age and results from an immature circulatory system completing the switch from fetal to extrauterine life. Letting the parent know that this is likely normal helps ease the parent's stress, but the healthcare provider must determine if this is actually what is occurring. Detailed descriptions or using medical terminology are not therapeutic communication techniques.

A 14-year-old male client is brought to the clinic by his parent with concerns that he is developing an excessive amount of breast tissue. The examination confirms that he has slight enlargement of the breast tissue. What information should be relayed to the adolescent and the parent? The adolescent will need to have hormonal levels assessed for the presence of estrogen. This is a normal and transient condition of adolescent males. This growth is abnormal but cannot be managed until his growth has stopped. Hormone therapy may be initiated to resolve the condition.

Rationale: Breast growth in adolescent males may occur in response to hormonal levels. This condition will self-resolve as hormones become more balanced. Therapy and laboratory studies are not indicated at this time.

The nurse is performing an abdominal assessment on a child. Which assessment techniques demonstrated by the nurse are appropriate when performing an abdominal assessment? (Select all that apply.) The nurse uses the technique of inspection to assess for the presence of an umbilical hernia. The nurse palpates the abdomen for the presence of any tenderness followed by auscultation of bowel sounds. The nurse uses percussion and notes dullness along the costal margins and tympany over the remainder of the abdomen. The nurse determines a child has absent bowel sounds after listening to each quadrant for 1 full minute. The nurse auscultates for bowel sounds in all four quadrants of the abdomen.

Rationale: Correct techniques include percussion of the abdomen to determine dullness along the costal margins and tympany over the remainder of the abdomen, inspection to visualize an umbilical hernia, and auscultation of all four quadrants of the abdomen. Auscultation should occur prior to palpation in order to not alter bowel sounds, and the absence of bowel sounds can only be determined when listening for at least 5 minutes in each quadrant.

The nurse has just finished administering the DTaP vaccine to a 2-month-old infant and is educating the parent about the immunization. Which statement is accurate? "Bring your infant back for the second dose when the infant is 4 months old." "There are no side effects from this vaccine." "You need to renew this immunization every 10 years." "The 'T' stands for tuberculosis."

Rationale: DTaP is given as a series of five injections—at 2, 4, and 6 months; between 15 and 18 months, and between 4 and 6 years. A TdaP booster is needed by 11 to 12 years. There are common side effects such as fever and redness and swelling at the injection site as well as other less common reactions such as seizures. The "T" in the vaccine stands for tetanus.

The student nurse is performing an assessment of an infant. Which action by the student nurse requires further instruction by the instructor? The student nurse asks the parents to step out of the room while performing the assessment. The student nurse examines the infant's ears after performing the head to toe assessment. The student nurse auscultates the infant's heart while the baby is sleeping. The student nurse undresses the infant except for the diaper prior to the assessment.

Rationale: It is often helpful for the parents to hold the infant during the assessment. This provides the infant with a sense of security during the assessment; therefore, it is not necessary for the parents to be asked to step out of the room during the assessment. Auscultation of the heart while the infant is sleeping allows a better assessment since the infant is not moving or crying. Assessing the ears often evokes crying so this should be left until the end of the assessment. Undressing the infant is necessary to perform a thorough assessment.

When preparing to examine a 2-year-old child, which action by the nurse will best establish rapport? Perform the examination prior to obtaining the health history to get it taken care of before the child becomes tired or impatient. Give the child a small toy to play with. Bend down to the child's eye level to establish contact. Ask the parent to let you hold the child prior to performing the examination.

Rationale: Making eye contact with the child is beneficial to establishing rapport prior to the examination. Toddlers may not appreciate being held by a stranger. The parents should be allowed to hold the child as much as possible during the examination to relay a sense of security to the child. A toy may be appreciated by the child but it does not promote a therapeutic rapport for the examination.

The parent of a 9-month-old child reports the child's eyes are often crossed. The nurse confirms this during the examination. What action is indicated? Ask the parent if this was a problem in siblings. Explain to the child's parent that this is normal until about 1 year of age. Document the finding as normal. Report the findings to the healthcare provider.

Rationale: Persistent strabismus is normal in newborns. If noted after the age of 6 months it should be evaluated by a pediatric ophthalmologist. This will need to be reported to the healthcare provider so that the referral can be made.

The nurse is preparing to assess the respiratory rate of a crying 15-month-old child. To get the most accurate assessment, what approach should the nurse take? Count the respiratory rate for 30 seconds. Place a stethoscope to count respirations. Count abdominal movements. Count after the child stops crying and is comfortable.

Rationale: Respirations should be assessed when the child is resting or sitting quietly because respiratory rate changes significantly when children cry, eat, or become more active. They also breathe more rapidly when anxious or frightened. Counting respirations for a full minute assures accuracy. Infants' respirations are primarily diaphragmatic; therefore, counting abdominal movements promotes accuracy. Placing a stethoscope to count respirations tends to be seen as invasive by a toddler and will result in movement away or an increase in respirations.

The school nurse is preparing a presentation for a group of teachers about teen suicide. When discussing risk factors for this occurrence which should be included? (Select all that apply.) Correctoption History of mental illness Unansweredoption Family history of depression Correctoption Homosexuality Unansweredoption Sexual promiscuity Correctoption Substance use disorder

Rationale: Suicide is the third leading cause of death in teens and young adults. There are identifiable risk factors that place a teen at an increased risk for suicide. These risk factors include substance use disorder, a history of mental illness, and homosexuality. A history of depression in the teen, not in family members, is considered a risk factor for suicide. Sexual promiscuity is not considered an increased risk for suicide.

The school nurse is monitoring a student athlete that experienced a concussion 2 weeks ago during a soccer game. The student reports having academic issues in a course in which the student previously had no difficulty. Which action should the nurse's take first? Immediately notify the student's parents that their child should see the healthcare provider Ask the student to describe the academic issues the student is having in the class Ask the student's teacher if the student is following the recommendations given to him or her for cognitive activity Perform a neurologic assessment on the student to see if there are any changes in neurologic status

Rationale: Talking with the teacher and performing a neurologic assessment are actions that may be necessary, but the nurse must first determine if the student is following the recommended level of cognitive activity; this can be accomplished by asking the child to describe the issues he or she is having. Contacting the parents immediately would not be warranted until sufficient information is collected.

What exercise recommendation should nurses give to adolescents? Engage in at least one competitive sport during the school year. Participate in physical activities that cause sweating and hard breathing weekly. Participate in 60 minutes of moderate to vigorous physical activity each day. Engage in aerobic exercise for 30 minutes 3 days per week.

Rationale: The U.S. Department of Health and Human Services recommends that adolescents participate in 60 minutes of moderate to vigorous physical activity each day. Aerobic exercise for 30 minutes three times weekly is not the recommended amount of exercise. The other recommendations lack adequate activity or specificity.

The nurse is talking with parents of a depressed 16-year-old child. Which question is of the most importance? Is there a gun in your home? Does your child exercise? How is your child's personal hygiene? Have your child's sleeping and eating habits changed?

Rationale: The child may be at risk for suicide. Firearm-related suicides have been responsible for a large number of the suicide deaths in 15- to 19-year-olds nationwide. All the other questions assess for depression and do not protect against suicide.

The nurse is obtaining a functional history during an admission assessment of a 12-year-old client. Which questions would be appropriate for the nurse to ask during this part of the assessment? (Select all that apply.) "Do you use a computer or a smart phone?" "Do you wear a seat belt any time you are a passenger in a car?" "Are your parents married?" "Can you tell me if you play any sports or participate in any physical activities?" "Do you know if your family has a history of any heart problems?"

Rationale: The functional history should contain information about the child's daily routine. Questions such as the amount of physical activity, car safety, and use of computers and smart phones (including the amount of time on these devices) are included in this assessment. Asking about heart problems is included in the family history assessment, and asking about parents is included in the family composition assessment

The nurse is planning to talk to a 12½-year-old boy who says, "I am too short! Girls are taller than I am. Guess I won't be playing basketball." What points of information will the nurse share? (Select all that apply.) "You can't control when and how much you will grow. Basketball can use some really fast guards." "You could add as much as 12 in to your height before you stop growing." "There are a lot of size differences in your age group now because everyone enters puberty and grows at slightly different times than others." "If you know how tall you were at age 3, you can double that and predict fairly well how tall you will get." "Girls begin to grow rapidly at a younger age than boys do. Boys start later and grow longer." "You are at the age now when most boys begin their growth spurt and grow very rapidly."

Rationale: The information about girls growing faster than boys, about differences in size within his age group, and about his growth potential is accurate and addresses concerns implied in his comments to the nurse. Most boys' growth spurts begin near 14 years, not 12½. Height should be doubled at 2 years to predict adult size. The growth control and basketball remarks minimize his concerns and provide no information.

The parents of a 3-week-old child question the presence of light pink "spots" on the back of their baby's neck. The nurse checks the areas and recognizes they are salmon nevi. What response by the nurse is most appropriate? "These are harmless and will gradually fade but may not go away." "These spots are small bruises that are from the delivery and just have not completely faded." "These are pediatric moles and will likely require laser therapy to resolve them once the child is older." "These are often called stork bites and will eventually be completely gone."

Rationale: The macules are often called stork bites or salmon nevi. They will fade but may never totally go away. Laser therapy is not needed to resolve them. The spots are not bruises related to labor and delivery.

The parent of a toddler tells the nurse that the child had a fever the previous night. During the assessment, which statement by the parent indicates further discussion is necessary regarding temperature measurement? "We have an electronic oral thermometer. It seemed to match our child's symptoms of fever better." "I know rectal temperature is pretty accurate but I didn't see that it was necessary to cause the discomfort of that route." "I used one of those thermometers that goes in the ear, but I don't think it was accurate." "My mother said she always used a glass thermometer when I was a kid and it was very accurate. Maybe that would be better."

Rationale: The nurse should address the comment about the use of a glass thermometer. These thermometers should be avoided since they contain mercury, which is toxic if the thermometer would break. Tympanic temperature measurement is dependent on several factors, so accuracy is sometimes questionable. Oral electronic thermometers are generally very accurate. Rectal temperatures are usually not necessary due to being invasive.

A 16-year-old adolescent is talking with the nurse at a local health clinic about skin care. Which comments by the adolescent does the nurse determine require additional conversation? (Select all that apply.) Unansweredoption "Our coach makes us wear sun-protective clothes when we practice outside on the weekends." Correctoption "The more exposure and burns I get now will toughen my skin so I won't get skin cancer when I'm older." Correctoption "My favorite time of day to be outside is the middle of the day, around noon." Correctoption "I only tan before going on spring break to get a base tan so I won't burn." Unansweredoption "My mom had melanoma so she always makes me wear a sunscreen with an SPF of 30."

Rationale: The nurse should further discuss comments that demonstrate incorrect information about sun exposure. Any exposure to tanning beds should be avoided to prevent skin cancer risks. Other risks for skin cancer include being in the sun between the times of 10:00 AM and 4:00 PM, and sun exposure and burns during childhood and adolescence. A minimum SPF of 15 should be used, so SPF 30 is good practice, as is wearing sun-protective clothing when outside during the day.

The nurse is preparing to assess the internal ear structures of a 3-year-old toddler. The toddler is resistant to the otoscope. How should the nurse respond? "Please sit still so I can see inside your ears." "May I please look inside your ears?" "Come, sit on this pretty, little red chair." "Let's see if I can find some puppies or kittens."

Rationale: The nurse should try to gain the toddler's cooperation by playing a funny pretend game using the "puppies or kittens" to engage the toddler. It is more likely the preschooler would prefer to sit on a parent's lap even though a red chair was offered. Politely asking the toddler to sit still is respectful but not likely to gain cooperation. Asking permission to look into the toddler's ear is an invitation for the toddler to answer "no."

The student nurse is assessing a 9-year-old's cardiovascular system. Which assessment technique should the nurse further discuss with the student? The student presses on the nailbeds of fingers and toes for a few seconds then quickly releases to assess capillary refill. The student uses the fingertips to palpate the chest for lifts and heaves or thrills. The student auscultates the heart at the third to fourth intercostal space just medial of the child's left midclavicular line. The student nurse palpates the left radial pulse, then palpates the right radial pulse for comparison.

Rationale: The point of maximum impulse (PMI) is lateral to the left midclavicular line at the fifth intercostal space in children ages 7 years and older. The student nurse is demonstrating auscultation of the PMI for a child under the age of 4 years.

A 16-year-old female adolescent reports to the nurse that she is trying to improve her diet to loose weight. When assessing her dietary intake for adequacy which findings indicate the need for further education? (Select all that apply.) 1,000 mg calcium intake per day Fat comprises about 40% of the daily intake Protein comprises approximately 30% of the daily dietary intake Average caloric intake is 2,500 per day Consumes 8--10 mg of iron each day Clear

Rationale: The recommended calcium intake for teens is 1,300 mg/day. The average caloric intake is approximately 2,000. An intake of 2,500 calories per day is likely excessive and should be reduced. Fat intake should compose 25% to 35% of the daily diet. Daily iron intake for teen girls should be approximately 15 mg per day.

A nursing student is asked to provide reasons it is important for the healthcare provider or nurse practitioner to have knowledge of the community in which the families and children seen in the practice live. Which reason(s) should the student provide? (Select all that apply.) The community can be a contributor to child-family health or a cause of illness. Knowing the community is necessary in developing appropriate health surveillance programs. Understanding the community promotes improved working relationships between families and healthcare providers. Awareness of agencies serving children results from knowing the community. Developing partnerships with community agencies facilitates overcoming barriers to care.

Rationale: These are all good reasons for a healthcare provider or nurse practitioner to know the community. Awareness of the strengths and limits within the community helps the healthcare provider or nurse practitioner better manage the health of the families served. When nurses develop partnerships with community agencies such as schools, churches, and ancillary health facilities, barriers to care can be overcome.

Parents comment that their child seemed more coordinated on the basketball floor at 12 years than now at 14. The nurse can include what information in the response? It is more difficult to keep focused at 14 years than at 12. Rapid and uneven growth of early adolescence (11 to 14 years) can interfere with coordination. This may be a reaction to adult authority. Hand--eye coordination is affected negatively by puberty.

Rationale: Uneven growth of soft tissues and bones during growth spurts can cause decreased coordination for boys. The age of 14 years is usually the time of peak height velocity (PHV). Mental focus and reaction to adult authority are not causes of poor coordination. Gains in hand—eye coordination are made during puberty and become precise by late adolescence.

The nurse is assessing the abdomen of a 3-year-old. Which finding should be reported immediately? Rounded abdomen Active bowel sounds Tympany over the abdomen Visible peristaltic waves

Rationale: Visible peristaltic waves are abnormal and require further evaluation. The other findings are considered normal for the child's age.

The student nurse is preparing to administer an immunization to an 18-month-old child under the direction of a registered nurse. Which actions by the student nurse indicate the need for the registered nurse to intervene? (Select all that apply.) The student selects a needle that is 5/8 in to perform a IM injection into the deltoid muscle. The student nurse prepares the triceps muscle for the IM injection. The student nurse reports plans to use a 20G needle for an IM deltoid injection. The student nurse reports that IM injections are to be avoided for children under the age of 2 years. The student nurse obtains a 23G needle to use for the injection.

Rationale: When administering an injection in a child who is 18 months of age the nurse may utilize both intramuscular (IM) or subcutaneous sites. Intramuscular sites include the deltoid muscle and anterior thigh. The triceps is used for subcutaneous injections. The appropriate needle length for an IM injection into the deltoid muscle is 5/8 in.

The nurse is obtaining the health history for a 9-year-old child who has been brought to the ambulatory care clinic with complaints of a backache. Which initial action by the nurse is most appropriate? Palpate the child's back while asking the severity of discomfort being experienced. Ask the child's parent about when the parent was first made aware of the discomfort. Ask the child when the pain started. Ask the child to demonstrate movements involving the back.

Rationale: When beginning the interview it is best to ask the child about the health complaints. If additional information is needed the parent should then be consulted. Palpating the back and asking the child to demonstrate movements takes place during the examination portion of the appointment and not the health history portion.

The nurse is planning to talk to a 12½-year-old boy who says, "I am too short! Girls are taller than I am. Guess I won't be playing basketball." What points of information will the nurse share? (Select all that apply.) Correctoption"There are a lot of size differences in your age group now because everyone enters puberty and grows at slightly different times than others." Unansweredoption"If you know how tall you were at age 3, you can double that and predict fairly well how tall you will get." Unansweredoption"You are at the age now when most boys begin their growth spurt and grow very rapidly." Unansweredoption"You can't control when and how much you will grow. Basketball can use some really fast guards." Correctoption"You could add as much as 12 in to your height before you stop growing." Correctoption"Girls begin to grow rapidly at a younger age than boys do. Boys start later and grow longer."

Rationale:The information about girls growing faster than boys, about differences in size within his age group, and about his growth potential is accurate and addresses concerns implied in his comments to the nurse. Most boys' growth spurts begin near 14 years, not 12½. Height should be doubled at 2 years to predict adult size. The growth control and basketball remarks minimize his concerns and provide no information.


संबंधित स्टडी सेट्स

AP Macroeconomics: Unit 5 Progress Check MCQ

View Set

11.1 anaerobic chemotrophs, 11. 5 aerobic chemoorganotrophs

View Set

Practice Test Questions - Final Exam

View Set

Ch. 8 Motivation: From Concepts to Applications

View Set

NSG 333 Chapter 15: Postpartum Adaptations

View Set